A complete picture of the lattice of subfields for a cyclotomic extension over $mathbb{Q}$.












5














Is there a good general purpose algorithm (batch of theorems) allowing one to determine the intermediate fields between $mathbb{Q}(zeta)$ and $mathbb{Q}$, where $zeta$ is some primitive root of unity?



Let $p$ be a prime. Consider the case where $zeta=zeta_{p}$ is a primitive $p$-th root of unity. Then the Galois extension is cyclic of order $p-1$ and $1,zeta,dots,zeta^{p-1}$ is a $mathbb{Q}$-basis for the extension. In this case for any subgroup $H$ of $G=mathbb{Z}/(p-1)$, by considering the sum $$alpha_H=sum_{sigmain H}sigmazeta,$$
we can observe that $alpha_H$ lies in the fixed field for $H$, and any automorphism $tau$ not in $H$ (note automorphisms are identified with subgroups of $mathbb{Z}/(p-1)$ in the natural way), $tau$ does not fix $alpha_H$. Therefore we can conclude that $mathbb{Q}(alpha_H)$ is the fixed field of $H$.



In this way we can get all of the intermediate fields of $zeta_p$ for all odd primes $p$.



We also have a theorem that says if we have $n=p^sq^t$, then $$text{Gal}(mathbb{Q}(zeta_n)/mathbb{Q})simeq text{Gal}(mathbb{Q}(zeta_{p^s})/mathbb{Q})timestext{Gal}(mathbb{Q}(zeta_{q^t})/mathbb{Q}).$$



So what I have yet to understand is



How can one generally find the intermediate fields between $mathbb{Q}(zeta_{p^s})$ and $mathbb{Q}$ for $sge 1$? I would like to also understand the case where $p=2, s>1$ though this might turn out to be a separate case.



EDIT: Even the case $n=pq$ is a little murky to me. Even given the isomorphism given by the Chinese Remainder Theorem I don't see a priori how to get all the "product" subfields. My idea is that you can consider the separate subfields under $text{Gal}(mathbb{Q}(zeta_{p})/mathbb{Q})$ and $text{Gal}(mathbb{Q}(zeta_{q})/mathbb{Q})$ separately and then consider the pairwise product of the generators of various subfields to see if you get anything new, but my idea is too inchoate.










share|cite|improve this question




















  • 1




    "Then the Galois extension is cyclic of order $p$": if it were the case, there would be no intermediate extensions. The extension actually has degree $varphi(p)=p-1$
    – M Turgeon
    Apr 15 '12 at 18:49












  • thanks for the catch, fixed
    – user21725
    Apr 15 '12 at 18:50










  • And thus the $mathbb{Q}$-basis would be $1,zeta,cdots, zeta^{p-2}$
    – Jing Zhang
    Oct 27 '13 at 9:18
















5














Is there a good general purpose algorithm (batch of theorems) allowing one to determine the intermediate fields between $mathbb{Q}(zeta)$ and $mathbb{Q}$, where $zeta$ is some primitive root of unity?



Let $p$ be a prime. Consider the case where $zeta=zeta_{p}$ is a primitive $p$-th root of unity. Then the Galois extension is cyclic of order $p-1$ and $1,zeta,dots,zeta^{p-1}$ is a $mathbb{Q}$-basis for the extension. In this case for any subgroup $H$ of $G=mathbb{Z}/(p-1)$, by considering the sum $$alpha_H=sum_{sigmain H}sigmazeta,$$
we can observe that $alpha_H$ lies in the fixed field for $H$, and any automorphism $tau$ not in $H$ (note automorphisms are identified with subgroups of $mathbb{Z}/(p-1)$ in the natural way), $tau$ does not fix $alpha_H$. Therefore we can conclude that $mathbb{Q}(alpha_H)$ is the fixed field of $H$.



In this way we can get all of the intermediate fields of $zeta_p$ for all odd primes $p$.



We also have a theorem that says if we have $n=p^sq^t$, then $$text{Gal}(mathbb{Q}(zeta_n)/mathbb{Q})simeq text{Gal}(mathbb{Q}(zeta_{p^s})/mathbb{Q})timestext{Gal}(mathbb{Q}(zeta_{q^t})/mathbb{Q}).$$



So what I have yet to understand is



How can one generally find the intermediate fields between $mathbb{Q}(zeta_{p^s})$ and $mathbb{Q}$ for $sge 1$? I would like to also understand the case where $p=2, s>1$ though this might turn out to be a separate case.



EDIT: Even the case $n=pq$ is a little murky to me. Even given the isomorphism given by the Chinese Remainder Theorem I don't see a priori how to get all the "product" subfields. My idea is that you can consider the separate subfields under $text{Gal}(mathbb{Q}(zeta_{p})/mathbb{Q})$ and $text{Gal}(mathbb{Q}(zeta_{q})/mathbb{Q})$ separately and then consider the pairwise product of the generators of various subfields to see if you get anything new, but my idea is too inchoate.










share|cite|improve this question




















  • 1




    "Then the Galois extension is cyclic of order $p$": if it were the case, there would be no intermediate extensions. The extension actually has degree $varphi(p)=p-1$
    – M Turgeon
    Apr 15 '12 at 18:49












  • thanks for the catch, fixed
    – user21725
    Apr 15 '12 at 18:50










  • And thus the $mathbb{Q}$-basis would be $1,zeta,cdots, zeta^{p-2}$
    – Jing Zhang
    Oct 27 '13 at 9:18














5












5








5


3





Is there a good general purpose algorithm (batch of theorems) allowing one to determine the intermediate fields between $mathbb{Q}(zeta)$ and $mathbb{Q}$, where $zeta$ is some primitive root of unity?



Let $p$ be a prime. Consider the case where $zeta=zeta_{p}$ is a primitive $p$-th root of unity. Then the Galois extension is cyclic of order $p-1$ and $1,zeta,dots,zeta^{p-1}$ is a $mathbb{Q}$-basis for the extension. In this case for any subgroup $H$ of $G=mathbb{Z}/(p-1)$, by considering the sum $$alpha_H=sum_{sigmain H}sigmazeta,$$
we can observe that $alpha_H$ lies in the fixed field for $H$, and any automorphism $tau$ not in $H$ (note automorphisms are identified with subgroups of $mathbb{Z}/(p-1)$ in the natural way), $tau$ does not fix $alpha_H$. Therefore we can conclude that $mathbb{Q}(alpha_H)$ is the fixed field of $H$.



In this way we can get all of the intermediate fields of $zeta_p$ for all odd primes $p$.



We also have a theorem that says if we have $n=p^sq^t$, then $$text{Gal}(mathbb{Q}(zeta_n)/mathbb{Q})simeq text{Gal}(mathbb{Q}(zeta_{p^s})/mathbb{Q})timestext{Gal}(mathbb{Q}(zeta_{q^t})/mathbb{Q}).$$



So what I have yet to understand is



How can one generally find the intermediate fields between $mathbb{Q}(zeta_{p^s})$ and $mathbb{Q}$ for $sge 1$? I would like to also understand the case where $p=2, s>1$ though this might turn out to be a separate case.



EDIT: Even the case $n=pq$ is a little murky to me. Even given the isomorphism given by the Chinese Remainder Theorem I don't see a priori how to get all the "product" subfields. My idea is that you can consider the separate subfields under $text{Gal}(mathbb{Q}(zeta_{p})/mathbb{Q})$ and $text{Gal}(mathbb{Q}(zeta_{q})/mathbb{Q})$ separately and then consider the pairwise product of the generators of various subfields to see if you get anything new, but my idea is too inchoate.










share|cite|improve this question















Is there a good general purpose algorithm (batch of theorems) allowing one to determine the intermediate fields between $mathbb{Q}(zeta)$ and $mathbb{Q}$, where $zeta$ is some primitive root of unity?



Let $p$ be a prime. Consider the case where $zeta=zeta_{p}$ is a primitive $p$-th root of unity. Then the Galois extension is cyclic of order $p-1$ and $1,zeta,dots,zeta^{p-1}$ is a $mathbb{Q}$-basis for the extension. In this case for any subgroup $H$ of $G=mathbb{Z}/(p-1)$, by considering the sum $$alpha_H=sum_{sigmain H}sigmazeta,$$
we can observe that $alpha_H$ lies in the fixed field for $H$, and any automorphism $tau$ not in $H$ (note automorphisms are identified with subgroups of $mathbb{Z}/(p-1)$ in the natural way), $tau$ does not fix $alpha_H$. Therefore we can conclude that $mathbb{Q}(alpha_H)$ is the fixed field of $H$.



In this way we can get all of the intermediate fields of $zeta_p$ for all odd primes $p$.



We also have a theorem that says if we have $n=p^sq^t$, then $$text{Gal}(mathbb{Q}(zeta_n)/mathbb{Q})simeq text{Gal}(mathbb{Q}(zeta_{p^s})/mathbb{Q})timestext{Gal}(mathbb{Q}(zeta_{q^t})/mathbb{Q}).$$



So what I have yet to understand is



How can one generally find the intermediate fields between $mathbb{Q}(zeta_{p^s})$ and $mathbb{Q}$ for $sge 1$? I would like to also understand the case where $p=2, s>1$ though this might turn out to be a separate case.



EDIT: Even the case $n=pq$ is a little murky to me. Even given the isomorphism given by the Chinese Remainder Theorem I don't see a priori how to get all the "product" subfields. My idea is that you can consider the separate subfields under $text{Gal}(mathbb{Q}(zeta_{p})/mathbb{Q})$ and $text{Gal}(mathbb{Q}(zeta_{q})/mathbb{Q})$ separately and then consider the pairwise product of the generators of various subfields to see if you get anything new, but my idea is too inchoate.







galois-theory






share|cite|improve this question















share|cite|improve this question













share|cite|improve this question




share|cite|improve this question








edited Apr 15 '12 at 18:49

























asked Apr 15 '12 at 18:26







user21725















  • 1




    "Then the Galois extension is cyclic of order $p$": if it were the case, there would be no intermediate extensions. The extension actually has degree $varphi(p)=p-1$
    – M Turgeon
    Apr 15 '12 at 18:49












  • thanks for the catch, fixed
    – user21725
    Apr 15 '12 at 18:50










  • And thus the $mathbb{Q}$-basis would be $1,zeta,cdots, zeta^{p-2}$
    – Jing Zhang
    Oct 27 '13 at 9:18














  • 1




    "Then the Galois extension is cyclic of order $p$": if it were the case, there would be no intermediate extensions. The extension actually has degree $varphi(p)=p-1$
    – M Turgeon
    Apr 15 '12 at 18:49












  • thanks for the catch, fixed
    – user21725
    Apr 15 '12 at 18:50










  • And thus the $mathbb{Q}$-basis would be $1,zeta,cdots, zeta^{p-2}$
    – Jing Zhang
    Oct 27 '13 at 9:18








1




1




"Then the Galois extension is cyclic of order $p$": if it were the case, there would be no intermediate extensions. The extension actually has degree $varphi(p)=p-1$
– M Turgeon
Apr 15 '12 at 18:49






"Then the Galois extension is cyclic of order $p$": if it were the case, there would be no intermediate extensions. The extension actually has degree $varphi(p)=p-1$
– M Turgeon
Apr 15 '12 at 18:49














thanks for the catch, fixed
– user21725
Apr 15 '12 at 18:50




thanks for the catch, fixed
– user21725
Apr 15 '12 at 18:50












And thus the $mathbb{Q}$-basis would be $1,zeta,cdots, zeta^{p-2}$
– Jing Zhang
Oct 27 '13 at 9:18




And thus the $mathbb{Q}$-basis would be $1,zeta,cdots, zeta^{p-2}$
– Jing Zhang
Oct 27 '13 at 9:18










2 Answers
2






active

oldest

votes


















2














If $q$ is a power of an odd prime $p$, then the multiplicative group of units in the ring ${bf Z}/q{bf Z}$ is cyclic of order $q-(q/p)$, and that's also the Galois group of ${bf Q}(zeta_q)$ over the rationals, so it seems to me that your construction for the prime case works.






share|cite|improve this answer





















  • Thanks for the observation, I didn't see this. I think I have the kind of answer I'm looking for, I'll try to write it up soon. Maybe my real question is for a certain kind of presentation of a general solution.
    – user21725
    Apr 17 '12 at 1:31



















0














Let $zeta$ be a primitive $p^s$-th root of unity for a prime $p$ and a positive integer $s$. Let $G = operatorname{Gal}(mathbb{Q}(zeta)/mathbb{Q}).$ Let
$$ eta = zeta + zeta^p + ... + zeta^{p^{s-1}}.$$
One can prove by induction on $s$ that the set $mathcal{B} = {g(eta) ; | ; g in G}$ is a basis of $mathbb{Q}(zeta)/mathbb{Q}$. Now for any $alpha in mathbb{Q}(zeta)$ let $H$ be the subgroup of $G$ fixing $alpha$. Define
$$ beta = sum_{sigma in H} sigma(eta).$$
Since $tau(beta) = beta$ for all $tau in H$, $mathbb{Q}(beta)$ is a subfield of $mathbb{Q}(alpha)$. We will now show by contradiction that for any $tau in G setminus H$ that $tau(beta) neq beta$. Assume there exists a $tau in G setminus H$ such that $tau(beta) = beta$. Since $mathcal{B}$ is a basis for $mathbb{Q}(zeta)/mathbb{Q}$, there must exist a $sigma in H$ such that $tau circ sigma(eta) = iota(eta)$ where $iota$ is the identity element of $G$. Then $tau = sigma^{-1} in H$, which contradicts our assumption. We conclude that for all $tau in G setminus H$ we have $tau(beta) neq beta$. Thus $mathbb{Q}(beta)$ contains $mathbb{Q}(alpha)$. This proves $mathbb{Q}(beta) = mathbb{Q}(alpha)$.



This shows that all subfields of $mathbb{Q}(zeta)$ can be constructed as $mathbb{Q}(beta)$ where $beta = sum_{sigma in H} sigma(zeta + zeta^p + ... + zeta^{p^{s-1}})$ for a subgroup $H$ of $operatorname{Gal}(mathbb{Q}(zeta)/mathbb{Q})$.



For example, if $zeta$ is a primitive 9-th root of unity, then $eta = zeta + zeta^3$. Since 2 is a quadratic nonresidue modulo 3, the Galois group $G$ is cyclic with generator $tau$ defined by $tau(zeta) = zeta^2$. Since $varphi(9) = 6$, the subgroups of $G$ are itself, the trivial subgroup, $left< tau^2 right>$, and $left<tau^3right>$. We know what fields we'll get for the first two from basic Galois theory, so now we must now find $beta = sum_{sigma in H}sigma(eta)$ for the two proper nontrivial subgroups. When $H = left<tau^2right>$



$$beta = sum_{sigma in left< tau^2 right>} sigma(eta) = (zeta + zeta^3) + (zeta^4 + zeta^3) + (zeta^7 + zeta^3) = 3zeta^3.$$
In this case $mathbb{Q}(beta) = mathbb{Q}(zeta^3)$. When $H = left< tau^3 right>$



$$beta = sum_{sigma in left< tau^3 right>} sigma(eta) = (zeta + zeta^3) + (zeta^8 + zeta^6) = zeta + zeta^8 - 1$$
In this case $mathbb{Q}(beta) = mathbb{Q}(zeta + zeta^8) = mathbb{Q} left( cos left(frac{2 pi}{9} right) right)$.









share|cite|improve this answer























    Your Answer





    StackExchange.ifUsing("editor", function () {
    return StackExchange.using("mathjaxEditing", function () {
    StackExchange.MarkdownEditor.creationCallbacks.add(function (editor, postfix) {
    StackExchange.mathjaxEditing.prepareWmdForMathJax(editor, postfix, [["$", "$"], ["\\(","\\)"]]);
    });
    });
    }, "mathjax-editing");

    StackExchange.ready(function() {
    var channelOptions = {
    tags: "".split(" "),
    id: "69"
    };
    initTagRenderer("".split(" "), "".split(" "), channelOptions);

    StackExchange.using("externalEditor", function() {
    // Have to fire editor after snippets, if snippets enabled
    if (StackExchange.settings.snippets.snippetsEnabled) {
    StackExchange.using("snippets", function() {
    createEditor();
    });
    }
    else {
    createEditor();
    }
    });

    function createEditor() {
    StackExchange.prepareEditor({
    heartbeatType: 'answer',
    autoActivateHeartbeat: false,
    convertImagesToLinks: true,
    noModals: true,
    showLowRepImageUploadWarning: true,
    reputationToPostImages: 10,
    bindNavPrevention: true,
    postfix: "",
    imageUploader: {
    brandingHtml: "Powered by u003ca class="icon-imgur-white" href="https://imgur.com/"u003eu003c/au003e",
    contentPolicyHtml: "User contributions licensed under u003ca href="https://creativecommons.org/licenses/by-sa/3.0/"u003ecc by-sa 3.0 with attribution requiredu003c/au003e u003ca href="https://stackoverflow.com/legal/content-policy"u003e(content policy)u003c/au003e",
    allowUrls: true
    },
    noCode: true, onDemand: true,
    discardSelector: ".discard-answer"
    ,immediatelyShowMarkdownHelp:true
    });


    }
    });














    draft saved

    draft discarded


















    StackExchange.ready(
    function () {
    StackExchange.openid.initPostLogin('.new-post-login', 'https%3a%2f%2fmath.stackexchange.com%2fquestions%2f132169%2fa-complete-picture-of-the-lattice-of-subfields-for-a-cyclotomic-extension-over%23new-answer', 'question_page');
    }
    );

    Post as a guest















    Required, but never shown
























    2 Answers
    2






    active

    oldest

    votes








    2 Answers
    2






    active

    oldest

    votes









    active

    oldest

    votes






    active

    oldest

    votes









    2














    If $q$ is a power of an odd prime $p$, then the multiplicative group of units in the ring ${bf Z}/q{bf Z}$ is cyclic of order $q-(q/p)$, and that's also the Galois group of ${bf Q}(zeta_q)$ over the rationals, so it seems to me that your construction for the prime case works.






    share|cite|improve this answer





















    • Thanks for the observation, I didn't see this. I think I have the kind of answer I'm looking for, I'll try to write it up soon. Maybe my real question is for a certain kind of presentation of a general solution.
      – user21725
      Apr 17 '12 at 1:31
















    2














    If $q$ is a power of an odd prime $p$, then the multiplicative group of units in the ring ${bf Z}/q{bf Z}$ is cyclic of order $q-(q/p)$, and that's also the Galois group of ${bf Q}(zeta_q)$ over the rationals, so it seems to me that your construction for the prime case works.






    share|cite|improve this answer





















    • Thanks for the observation, I didn't see this. I think I have the kind of answer I'm looking for, I'll try to write it up soon. Maybe my real question is for a certain kind of presentation of a general solution.
      – user21725
      Apr 17 '12 at 1:31














    2












    2








    2






    If $q$ is a power of an odd prime $p$, then the multiplicative group of units in the ring ${bf Z}/q{bf Z}$ is cyclic of order $q-(q/p)$, and that's also the Galois group of ${bf Q}(zeta_q)$ over the rationals, so it seems to me that your construction for the prime case works.






    share|cite|improve this answer












    If $q$ is a power of an odd prime $p$, then the multiplicative group of units in the ring ${bf Z}/q{bf Z}$ is cyclic of order $q-(q/p)$, and that's also the Galois group of ${bf Q}(zeta_q)$ over the rationals, so it seems to me that your construction for the prime case works.







    share|cite|improve this answer












    share|cite|improve this answer



    share|cite|improve this answer










    answered Apr 16 '12 at 0:22









    Gerry MyersonGerry Myerson

    146k8147298




    146k8147298












    • Thanks for the observation, I didn't see this. I think I have the kind of answer I'm looking for, I'll try to write it up soon. Maybe my real question is for a certain kind of presentation of a general solution.
      – user21725
      Apr 17 '12 at 1:31


















    • Thanks for the observation, I didn't see this. I think I have the kind of answer I'm looking for, I'll try to write it up soon. Maybe my real question is for a certain kind of presentation of a general solution.
      – user21725
      Apr 17 '12 at 1:31
















    Thanks for the observation, I didn't see this. I think I have the kind of answer I'm looking for, I'll try to write it up soon. Maybe my real question is for a certain kind of presentation of a general solution.
    – user21725
    Apr 17 '12 at 1:31




    Thanks for the observation, I didn't see this. I think I have the kind of answer I'm looking for, I'll try to write it up soon. Maybe my real question is for a certain kind of presentation of a general solution.
    – user21725
    Apr 17 '12 at 1:31











    0














    Let $zeta$ be a primitive $p^s$-th root of unity for a prime $p$ and a positive integer $s$. Let $G = operatorname{Gal}(mathbb{Q}(zeta)/mathbb{Q}).$ Let
    $$ eta = zeta + zeta^p + ... + zeta^{p^{s-1}}.$$
    One can prove by induction on $s$ that the set $mathcal{B} = {g(eta) ; | ; g in G}$ is a basis of $mathbb{Q}(zeta)/mathbb{Q}$. Now for any $alpha in mathbb{Q}(zeta)$ let $H$ be the subgroup of $G$ fixing $alpha$. Define
    $$ beta = sum_{sigma in H} sigma(eta).$$
    Since $tau(beta) = beta$ for all $tau in H$, $mathbb{Q}(beta)$ is a subfield of $mathbb{Q}(alpha)$. We will now show by contradiction that for any $tau in G setminus H$ that $tau(beta) neq beta$. Assume there exists a $tau in G setminus H$ such that $tau(beta) = beta$. Since $mathcal{B}$ is a basis for $mathbb{Q}(zeta)/mathbb{Q}$, there must exist a $sigma in H$ such that $tau circ sigma(eta) = iota(eta)$ where $iota$ is the identity element of $G$. Then $tau = sigma^{-1} in H$, which contradicts our assumption. We conclude that for all $tau in G setminus H$ we have $tau(beta) neq beta$. Thus $mathbb{Q}(beta)$ contains $mathbb{Q}(alpha)$. This proves $mathbb{Q}(beta) = mathbb{Q}(alpha)$.



    This shows that all subfields of $mathbb{Q}(zeta)$ can be constructed as $mathbb{Q}(beta)$ where $beta = sum_{sigma in H} sigma(zeta + zeta^p + ... + zeta^{p^{s-1}})$ for a subgroup $H$ of $operatorname{Gal}(mathbb{Q}(zeta)/mathbb{Q})$.



    For example, if $zeta$ is a primitive 9-th root of unity, then $eta = zeta + zeta^3$. Since 2 is a quadratic nonresidue modulo 3, the Galois group $G$ is cyclic with generator $tau$ defined by $tau(zeta) = zeta^2$. Since $varphi(9) = 6$, the subgroups of $G$ are itself, the trivial subgroup, $left< tau^2 right>$, and $left<tau^3right>$. We know what fields we'll get for the first two from basic Galois theory, so now we must now find $beta = sum_{sigma in H}sigma(eta)$ for the two proper nontrivial subgroups. When $H = left<tau^2right>$



    $$beta = sum_{sigma in left< tau^2 right>} sigma(eta) = (zeta + zeta^3) + (zeta^4 + zeta^3) + (zeta^7 + zeta^3) = 3zeta^3.$$
    In this case $mathbb{Q}(beta) = mathbb{Q}(zeta^3)$. When $H = left< tau^3 right>$



    $$beta = sum_{sigma in left< tau^3 right>} sigma(eta) = (zeta + zeta^3) + (zeta^8 + zeta^6) = zeta + zeta^8 - 1$$
    In this case $mathbb{Q}(beta) = mathbb{Q}(zeta + zeta^8) = mathbb{Q} left( cos left(frac{2 pi}{9} right) right)$.









    share|cite|improve this answer




























      0














      Let $zeta$ be a primitive $p^s$-th root of unity for a prime $p$ and a positive integer $s$. Let $G = operatorname{Gal}(mathbb{Q}(zeta)/mathbb{Q}).$ Let
      $$ eta = zeta + zeta^p + ... + zeta^{p^{s-1}}.$$
      One can prove by induction on $s$ that the set $mathcal{B} = {g(eta) ; | ; g in G}$ is a basis of $mathbb{Q}(zeta)/mathbb{Q}$. Now for any $alpha in mathbb{Q}(zeta)$ let $H$ be the subgroup of $G$ fixing $alpha$. Define
      $$ beta = sum_{sigma in H} sigma(eta).$$
      Since $tau(beta) = beta$ for all $tau in H$, $mathbb{Q}(beta)$ is a subfield of $mathbb{Q}(alpha)$. We will now show by contradiction that for any $tau in G setminus H$ that $tau(beta) neq beta$. Assume there exists a $tau in G setminus H$ such that $tau(beta) = beta$. Since $mathcal{B}$ is a basis for $mathbb{Q}(zeta)/mathbb{Q}$, there must exist a $sigma in H$ such that $tau circ sigma(eta) = iota(eta)$ where $iota$ is the identity element of $G$. Then $tau = sigma^{-1} in H$, which contradicts our assumption. We conclude that for all $tau in G setminus H$ we have $tau(beta) neq beta$. Thus $mathbb{Q}(beta)$ contains $mathbb{Q}(alpha)$. This proves $mathbb{Q}(beta) = mathbb{Q}(alpha)$.



      This shows that all subfields of $mathbb{Q}(zeta)$ can be constructed as $mathbb{Q}(beta)$ where $beta = sum_{sigma in H} sigma(zeta + zeta^p + ... + zeta^{p^{s-1}})$ for a subgroup $H$ of $operatorname{Gal}(mathbb{Q}(zeta)/mathbb{Q})$.



      For example, if $zeta$ is a primitive 9-th root of unity, then $eta = zeta + zeta^3$. Since 2 is a quadratic nonresidue modulo 3, the Galois group $G$ is cyclic with generator $tau$ defined by $tau(zeta) = zeta^2$. Since $varphi(9) = 6$, the subgroups of $G$ are itself, the trivial subgroup, $left< tau^2 right>$, and $left<tau^3right>$. We know what fields we'll get for the first two from basic Galois theory, so now we must now find $beta = sum_{sigma in H}sigma(eta)$ for the two proper nontrivial subgroups. When $H = left<tau^2right>$



      $$beta = sum_{sigma in left< tau^2 right>} sigma(eta) = (zeta + zeta^3) + (zeta^4 + zeta^3) + (zeta^7 + zeta^3) = 3zeta^3.$$
      In this case $mathbb{Q}(beta) = mathbb{Q}(zeta^3)$. When $H = left< tau^3 right>$



      $$beta = sum_{sigma in left< tau^3 right>} sigma(eta) = (zeta + zeta^3) + (zeta^8 + zeta^6) = zeta + zeta^8 - 1$$
      In this case $mathbb{Q}(beta) = mathbb{Q}(zeta + zeta^8) = mathbb{Q} left( cos left(frac{2 pi}{9} right) right)$.









      share|cite|improve this answer


























        0












        0








        0






        Let $zeta$ be a primitive $p^s$-th root of unity for a prime $p$ and a positive integer $s$. Let $G = operatorname{Gal}(mathbb{Q}(zeta)/mathbb{Q}).$ Let
        $$ eta = zeta + zeta^p + ... + zeta^{p^{s-1}}.$$
        One can prove by induction on $s$ that the set $mathcal{B} = {g(eta) ; | ; g in G}$ is a basis of $mathbb{Q}(zeta)/mathbb{Q}$. Now for any $alpha in mathbb{Q}(zeta)$ let $H$ be the subgroup of $G$ fixing $alpha$. Define
        $$ beta = sum_{sigma in H} sigma(eta).$$
        Since $tau(beta) = beta$ for all $tau in H$, $mathbb{Q}(beta)$ is a subfield of $mathbb{Q}(alpha)$. We will now show by contradiction that for any $tau in G setminus H$ that $tau(beta) neq beta$. Assume there exists a $tau in G setminus H$ such that $tau(beta) = beta$. Since $mathcal{B}$ is a basis for $mathbb{Q}(zeta)/mathbb{Q}$, there must exist a $sigma in H$ such that $tau circ sigma(eta) = iota(eta)$ where $iota$ is the identity element of $G$. Then $tau = sigma^{-1} in H$, which contradicts our assumption. We conclude that for all $tau in G setminus H$ we have $tau(beta) neq beta$. Thus $mathbb{Q}(beta)$ contains $mathbb{Q}(alpha)$. This proves $mathbb{Q}(beta) = mathbb{Q}(alpha)$.



        This shows that all subfields of $mathbb{Q}(zeta)$ can be constructed as $mathbb{Q}(beta)$ where $beta = sum_{sigma in H} sigma(zeta + zeta^p + ... + zeta^{p^{s-1}})$ for a subgroup $H$ of $operatorname{Gal}(mathbb{Q}(zeta)/mathbb{Q})$.



        For example, if $zeta$ is a primitive 9-th root of unity, then $eta = zeta + zeta^3$. Since 2 is a quadratic nonresidue modulo 3, the Galois group $G$ is cyclic with generator $tau$ defined by $tau(zeta) = zeta^2$. Since $varphi(9) = 6$, the subgroups of $G$ are itself, the trivial subgroup, $left< tau^2 right>$, and $left<tau^3right>$. We know what fields we'll get for the first two from basic Galois theory, so now we must now find $beta = sum_{sigma in H}sigma(eta)$ for the two proper nontrivial subgroups. When $H = left<tau^2right>$



        $$beta = sum_{sigma in left< tau^2 right>} sigma(eta) = (zeta + zeta^3) + (zeta^4 + zeta^3) + (zeta^7 + zeta^3) = 3zeta^3.$$
        In this case $mathbb{Q}(beta) = mathbb{Q}(zeta^3)$. When $H = left< tau^3 right>$



        $$beta = sum_{sigma in left< tau^3 right>} sigma(eta) = (zeta + zeta^3) + (zeta^8 + zeta^6) = zeta + zeta^8 - 1$$
        In this case $mathbb{Q}(beta) = mathbb{Q}(zeta + zeta^8) = mathbb{Q} left( cos left(frac{2 pi}{9} right) right)$.









        share|cite|improve this answer














        Let $zeta$ be a primitive $p^s$-th root of unity for a prime $p$ and a positive integer $s$. Let $G = operatorname{Gal}(mathbb{Q}(zeta)/mathbb{Q}).$ Let
        $$ eta = zeta + zeta^p + ... + zeta^{p^{s-1}}.$$
        One can prove by induction on $s$ that the set $mathcal{B} = {g(eta) ; | ; g in G}$ is a basis of $mathbb{Q}(zeta)/mathbb{Q}$. Now for any $alpha in mathbb{Q}(zeta)$ let $H$ be the subgroup of $G$ fixing $alpha$. Define
        $$ beta = sum_{sigma in H} sigma(eta).$$
        Since $tau(beta) = beta$ for all $tau in H$, $mathbb{Q}(beta)$ is a subfield of $mathbb{Q}(alpha)$. We will now show by contradiction that for any $tau in G setminus H$ that $tau(beta) neq beta$. Assume there exists a $tau in G setminus H$ such that $tau(beta) = beta$. Since $mathcal{B}$ is a basis for $mathbb{Q}(zeta)/mathbb{Q}$, there must exist a $sigma in H$ such that $tau circ sigma(eta) = iota(eta)$ where $iota$ is the identity element of $G$. Then $tau = sigma^{-1} in H$, which contradicts our assumption. We conclude that for all $tau in G setminus H$ we have $tau(beta) neq beta$. Thus $mathbb{Q}(beta)$ contains $mathbb{Q}(alpha)$. This proves $mathbb{Q}(beta) = mathbb{Q}(alpha)$.



        This shows that all subfields of $mathbb{Q}(zeta)$ can be constructed as $mathbb{Q}(beta)$ where $beta = sum_{sigma in H} sigma(zeta + zeta^p + ... + zeta^{p^{s-1}})$ for a subgroup $H$ of $operatorname{Gal}(mathbb{Q}(zeta)/mathbb{Q})$.



        For example, if $zeta$ is a primitive 9-th root of unity, then $eta = zeta + zeta^3$. Since 2 is a quadratic nonresidue modulo 3, the Galois group $G$ is cyclic with generator $tau$ defined by $tau(zeta) = zeta^2$. Since $varphi(9) = 6$, the subgroups of $G$ are itself, the trivial subgroup, $left< tau^2 right>$, and $left<tau^3right>$. We know what fields we'll get for the first two from basic Galois theory, so now we must now find $beta = sum_{sigma in H}sigma(eta)$ for the two proper nontrivial subgroups. When $H = left<tau^2right>$



        $$beta = sum_{sigma in left< tau^2 right>} sigma(eta) = (zeta + zeta^3) + (zeta^4 + zeta^3) + (zeta^7 + zeta^3) = 3zeta^3.$$
        In this case $mathbb{Q}(beta) = mathbb{Q}(zeta^3)$. When $H = left< tau^3 right>$



        $$beta = sum_{sigma in left< tau^3 right>} sigma(eta) = (zeta + zeta^3) + (zeta^8 + zeta^6) = zeta + zeta^8 - 1$$
        In this case $mathbb{Q}(beta) = mathbb{Q}(zeta + zeta^8) = mathbb{Q} left( cos left(frac{2 pi}{9} right) right)$.










        share|cite|improve this answer














        share|cite|improve this answer



        share|cite|improve this answer








        edited Sep 28 '17 at 19:39

























        answered Jun 26 '17 at 16:02









        Ryan T JohnsonRyan T Johnson

        914




        914






























            draft saved

            draft discarded




















































            Thanks for contributing an answer to Mathematics Stack Exchange!


            • Please be sure to answer the question. Provide details and share your research!

            But avoid



            • Asking for help, clarification, or responding to other answers.

            • Making statements based on opinion; back them up with references or personal experience.


            Use MathJax to format equations. MathJax reference.


            To learn more, see our tips on writing great answers.





            Some of your past answers have not been well-received, and you're in danger of being blocked from answering.


            Please pay close attention to the following guidance:


            • Please be sure to answer the question. Provide details and share your research!

            But avoid



            • Asking for help, clarification, or responding to other answers.

            • Making statements based on opinion; back them up with references or personal experience.


            To learn more, see our tips on writing great answers.




            draft saved


            draft discarded














            StackExchange.ready(
            function () {
            StackExchange.openid.initPostLogin('.new-post-login', 'https%3a%2f%2fmath.stackexchange.com%2fquestions%2f132169%2fa-complete-picture-of-the-lattice-of-subfields-for-a-cyclotomic-extension-over%23new-answer', 'question_page');
            }
            );

            Post as a guest















            Required, but never shown





















































            Required, but never shown














            Required, but never shown












            Required, but never shown







            Required, but never shown

































            Required, but never shown














            Required, but never shown












            Required, but never shown







            Required, but never shown







            Popular posts from this blog

            Human spaceflight

            Can not write log (Is /dev/pts mounted?) - openpty in Ubuntu-on-Windows?

            File:DeusFollowingSea.jpg